MayMay
Thanks Received: 0
Forum Guests
 
Posts: 25
Joined: January 02nd, 2013
 
 
 

Q15 - letter to the editor

by MayMay Tue Nov 26, 2013 6:25 pm

the conclusion is that the proposal is justified....even though it's a form of stealing.


The answer choice talks about motivation. there is nothing in the stimulus about motivation. If I pick this answer choice, i am to assume that the motive of compensation is good. Is that a valid assumption? during the test, I didn't think it was...
User avatar
 
ohthatpatrick
Thanks Received: 3808
Atticus Finch
Atticus Finch
 
Posts: 4661
Joined: April 01st, 2011
 
This post thanked 1 time.
 
 

Re: Q15 - letter to the editor

by ohthatpatrick Wed Nov 27, 2013 6:29 pm

"the motive" for confiscating a burglar's wages is "in order to fund an account to compensate burglary victims."

The purpose of an action could be called its motivation.

I don't think we actually need to assume that compensating burglary victims is "good" for (C) to strengthen the argument.

It's enough to see that the author is granting his opposition that even if confiscating wages could be considered stealing, it would still be justified. He's sort of saying, "whether or not we consider this action stealing, it's justified."

So he's assuming that 'the label' we assign to an action does not determine whether or not the action is justified.

What DOES make the proposal justified? We know the author's reason (premise) comes right before the conclusion ... the confiscating would be performed in order to fund an account for victims.

So (C) fits the structure of this argument, whether we agree that the motive is good or bad.

Usually, correct answers to Principle-Support are conditional and do a pretty good job of proving the truth of the conclusion.

This is one of those atypical correct answers that just strengthens the argument.

== other answers ==
(A) This is not a rule that helps us strengthen a conclusion that says "the proposal would still be justified". This is spelling out implicit logic of the proposal ... why are we confiscating wages and putting it into a fund for victims? Because money from a burglar should go to that burglar's victims.

But we're not trying to justify the logic of the proposal. We're trying to justify this author's argument: "Because the confiscated wages would go to a victims' fund, this confiscating is justified, whether or not we call it 'stealing'."

(B) This sounds incredibly close to (A), and all the same reasoning applies for shooting it down.

The #1 way we can get rid of answers on Principle-Support is to see whether they help us prove the conclusion.

The conclusion is "this proposal would still be justified". The principle in (B) is not a rule that helps prove "it is justified".

(D) This is backwards logic, although it's also talking about a crime being justified, and we can't quite call the government's proposal 'a crime'. But assuming we could, this would look like:
IF it's justified THEN it compensates ppl who deserve compensation

The actual argument flows from Prem to Conc as
IF it compensates ppl who deserve compensation THEN it's justified

(E) This weakens the argument.

Hope this helps.
 
gaheexlee
Thanks Received: 10
Elle Woods
Elle Woods
 
Posts: 55
Joined: May 27th, 2014
 
 
 

Re: Q15 - letter to the editor

by gaheexlee Mon Dec 01, 2014 12:20 pm

I picked the correct answer but had a question about it.

While (C) does link "motive" with "justified" to say that one's motives can determine whether or not something is justified, when applied to this argument, doesn't it still leave room for the argument to not be justified?

If for instance (C) had said "the motives always justify the action," then the conclusion would be guaranteed. But (C) as stated doesn't allow us to draw the conclusion that the government's action is in fact justified unless we ourselves decide that this particular motive does justify.

Is this vagueness condoned because this is a Principle-Support and not a Sufficient Assumption? I know the MLSAT book differentiates between the two and says the former doesn't have to 100% guarantee the conclusion.
User avatar
 
ohthatpatrick
Thanks Received: 3808
Atticus Finch
Atticus Finch
 
Posts: 4661
Joined: April 01st, 2011
 
This post thanked 1 time.
 
 

Re: Q15 - letter to the editor

by ohthatpatrick Thu Dec 04, 2014 4:31 pm

Yup, you nailed it!

The question stem asks us to pick the choice that most helps support the argument (whereas Sufficient Assumption demands that the answer choice makes the conclusion follow logically).

We have never established that "funding an account to compensate burglary victims" is a worthy, justified motive.

So the argument, with (C) added onto it, is still loose.

But (C) made it better.

Although most Principle-Support correct answers act like Sufficient Assumptions, they don't NEED to. The way the question stem is worded, we're really just Strengthening the argument.
 
judaydaday
Thanks Received: 6
Jackie Chiles
Jackie Chiles
 
Posts: 40
Joined: January 14th, 2015
 
This post thanked 2 times.
 
 

Re: Q15 - letter to the editor

by judaydaday Tue Jan 20, 2015 1:39 pm

Can someone verify if my line of thinking was correct?

I honestly just eliminated (A) and (B) because they were too narrow in scope. Since this is a Principle Q, the answer is supposed to be more general.

However, in (A) the AC specifies that the money should be given "to that burglar's victims."

Similarly, in (B) the AC specifies that the compensation should be given to the "same individuals they victimized."

Since the conclusion of the argument was just funding an account in general, (A) and (B) could be eliminated on the above mentioned basis. Is this correct?
User avatar
 
maryadkins
Thanks Received: 641
Atticus Finch
Atticus Finch
 
Posts: 1261
Joined: March 23rd, 2011
 
 
 

Re: Q15 - letter to the editor

by maryadkins Sun Jan 25, 2015 2:02 pm

Yes, I think that works! Good job.